Vector Calculus - Laplacian on Scalar Field

Click For Summary
The discussion focuses on calculating the second partial derivative of a scalar field \(\psi\) that depends on the distance \(r\) from the origin. The user has successfully applied the chain and product rules to derive the first derivative, but struggles with the manipulation of the second term in the second derivative. A suggestion is made to simplify the expression by rewriting it, which facilitates the application of the product and quotient rules. This guidance helps the user progress in their calculations. The conversation emphasizes the importance of correctly applying calculus rules in vector calculus problems.
Insolite
Messages
7
Reaction score
0
A scalar field \psi is dependent only on the distance r = \sqrt{x^{2} + y^{2} + z^{2}} from the origin.

Show:

\partial_{x}^{2}\psi = \left(\frac{1}{r} - \frac{x^{2}}{r^{3}}\right)\frac{d\psi}{dr} + \frac{x^{2}}{r^{2}}\frac{d^{2}\psi}{dr^{2}}

I've used the chain and product rules so far, but I'm unsure how to approach the problem to move onwards from this point. My working is as follows:

\frac{\partial\psi}{\partial x} = \frac{\partial r}{\partial x}\frac{d\psi}{dr} where the identity \frac{\partial r}{\partial x} = \frac{x}{r} is given.
\frac{\partial\psi}{\partial x} = \frac{x}{r}\frac{d\psi}{dr} (1)
\frac{\partial^{2}\psi}{\partial^{2}x} = \frac{\partial}{\partial x}\left(\frac{x}{r}\frac{d\psi}{dr}\right)
= \left(\frac{\partial}{\partial x}\frac{x}{r}\right)\left(\frac{d\psi}{dr}\right) + \left(\frac{\partial}{\partial x}\frac{d\psi}{dr}\right)\left(\frac{x}{r}\right)

At this point, I can solve the left term to give me:
-\frac{x^{2}}{r^{3}}\frac{d\psi}{dr}

But I don't know how to properly manipulate the second term. I've tried re-arranging and substituting in (1), but this didn't work out. Any hints on how to proceed would be greatly appreciated.

Thanks.
 
Physics news on Phys.org
Insolite said:
A scalar field \psi is dependent only on the distance r = \sqrt{x^{2} + y^{2} + z^{2}} from the origin.

Show:

\partial_{x}^{2}\psi = \left(\frac{1}{r} - \frac{x^{2}}{r^{3}}\right)\frac{d\psi}{dr} + \frac{x^{2}}{r^{2}}\frac{d^{2}\psi}{dr^{2}}

I've used the chain and product rules so far, but I'm unsure how to approach the problem to move onwards from this point. My working is as follows:

\frac{\partial\psi}{\partial x} = \frac{\partial r}{\partial x}\frac{d\psi}{dr} where the identity \frac{\partial r}{\partial x} = \frac{x}{r} is given.
\frac{\partial\psi}{\partial x} = \frac{x}{r}\frac{d\psi}{dr} (1)

Good so far.

\frac{\partial^{2}\psi}{\partial^{2}x} = \frac{\partial}{\partial x}\left(\frac{x}{r}\frac{d\psi}{dr}\right)
= \left(\frac{\partial}{\partial x}\frac{x}{r}\right)\left(\frac{d\psi}{dr}\right) + \left(\frac{\partial}{\partial x}\frac{d\psi}{dr}\right)\left(\frac{x}{r}\right)

You might find it easier to write it as ##\frac \partial {\partial x} (x\cdot \frac{\psi'(r} r)##. Now you have a product rule and when you calculate ##\frac \partial {\partial x}## of the second term (quotient rule), just use the chain rule like you did in the beginning.
 
Aah, thanks a lot for your help, I've got it now. (:
 
Question: A clock's minute hand has length 4 and its hour hand has length 3. What is the distance between the tips at the moment when it is increasing most rapidly?(Putnam Exam Question) Answer: Making assumption that both the hands moves at constant angular velocities, the answer is ## \sqrt{7} .## But don't you think this assumption is somewhat doubtful and wrong?

Similar threads

  • · Replies 1 ·
Replies
1
Views
1K
Replies
2
Views
1K
  • · Replies 7 ·
Replies
7
Views
2K
Replies
3
Views
2K
Replies
4
Views
1K
Replies
6
Views
1K
  • · Replies 5 ·
Replies
5
Views
1K
  • · Replies 4 ·
Replies
4
Views
2K
Replies
5
Views
2K
  • · Replies 3 ·
Replies
3
Views
3K